0 Daumen
1,9k Aufrufe
Ein Roboter A marschiert aus 25 m Entfernung mit einer Geschwindigkeit von 3m/min geradlinig auf eine Kreuzung zu. Gleichzeitig marschiert ein Roboter B auf einem dazu senkrechten Weg aus 30 m Entfernung mit einer Geschwindigkeit von 4m/min auf die selbe Kreuzung zu.

a) Zu welchem Zeitpunkt ist ihre Luftlinienentfernung am kleinsten?

b) Um wie viel Prozent müsste die Geschwindigkeit von Roboter A höher sein, damit die beiden Roboter an der Kreuzung zusammentreffen?

Ich verstehe die Aufgabe einfach nicht :(   Wir haben es immer so gemacht, dass wir zwei Gleichungen aufstellen, dann eine Variable ersetzen und die Nullstellen berechnen und so auf den Scheitel kommen.... Wenn möglich bitte bei eurem Rechenweg auch so machen...
Avatar von

2 Antworten

0 Daumen
 
Beste Antwort

Wenn mann keine Idee hat sollte man sich zunächst eine Skizze machen.

Ein Roboter A marschiert aus 25 m Entfernung mit einer Geschwindigkeit von 3m/min geradlinig auf eine Kreuzung zu. Gleichzeitig marschiert ein Roboter B auf einem dazu senkrechten Weg aus 30 m Entfernung und mit einer Geschwindigkeit von 4m/min auf die gleiche Kreuzung zu.

A: x(t) = -25 + 3*t
B: y(t) = -30 + 4*t

a) Zu welchem Zeitpunkt ist ihre Luftlienienentfernung  am kleinsten?

d^2 = (-25 + 3*t)^2 + (-30 + 4*t)^2
d^2 = 25·t^2 - 390·t + 1525
d^2' = 50·t - 390 = 0
t = 7.8 min

Nach 7.8 min haben sie den kleinsten Abstand.

b) Um wie viel Prozent müsste die Geschwindigkeit von Roboter A höher sein, damit die beiden Roboter an der Kreuzung zusammentreffen?

y(t) = -30 + 4*t = 0
t = 7.5

-25 + v*7.5 = 0
v = 10/3

10/3 / 3 - 1 = 0.1111111111 = 11.11%

Die Geschwindigkeit muss um 11.11% zunehmen.

Avatar von 479 k 🚀
Könntest du mir erklären, wie man bei der b) auf den Ansatz kommt? Was muss man sich denken?
Man muss sich fragen nach wie vielen Minuten Roboter B an der Kreuzung ist.
Dann muss man sich fragen Welche Geschwindigkeit Roboter A haben muss um zur selben Zeit an der Kreuzung zu sein.

Als letztes Fragt man sich um wie viel Prozent die Geschwindigkeit dann höher sein muss.

Genau diese drei Fragen habe ich oben mit Rechnung beantwortet.

Eine Frage:

Bei a) wie kommst du bei der Rechnung von Zeile 1 auf 2.Ich kapiere nicht so recht wie man das ausrechnet.Wär sehr nett wenn mir das jemand erklären würde :D

Meinst du

d2 = (-25 + 3*t)2 + (-30 + 4*t)2 
d2 = 25·t2 - 390·t + 1525 

Rechte Seite ausmultiplizieren und zusammenfassen. Stell sonst mal deine Rechnung rein. Dank an die binomischen Formeln.

Hi ist bisschen älter schon aber könnte mir jemand erklären warum auf einmal 50t da ist und die 1525 weg sind? Den Rest hab ich verstanden.


Hallo Einstein,

es wurde die erste Ableitung gebildet.

Gruß

Silvia

Danke für die schnelle Antwort. Wir fangen grade erst mit den extremwertaufgaben an und ich verstehe wieso da 25t2 - 390t + 1525 steht aber weiter weiß ich nicht wie ich verfahren soll.

Könntest du mir das kurz erklären?

Bei Extremwertaufgaben geht es darum, die Nebenbedingung so in die Zielfunktion einzusetzen, dass man nur noch eine Variable hat. Das war gegeben durch

d^2 = 25t^2 -390t + 1525

Da bei Extremwertaufgaben immer etwas Maximales oder Minimales gesucht wird, suchst du von der Funktion die Extremstellen. Dazu musst du erst einmal die erste Ableitung gleich null setzen und nach x auflösen.

In diesem Fall ergibt das

50t - 390 = 0            |+390, dann : 50

t = 7,8

Um zu überprüfen, ob es sich um ein Minimum oder Maximum handelt, setzt du den Wert in die zweite Ableitung ein.

d2''  = 50 > 0 ⇒ Minimum

Super vielen dank jetzt hab ich's verstanden. Gar nicht so schwer.

+1 Daumen

Eine Zeichnung sagt doch manchmal mehr als tausend Worte.

Avatar von 1,8 k

Luftlinienabstand (Pythagoras):
A(t)^2+B(t)^2=(25-3t)^2+(30-4t)^2

Ein anderes Problem?

Stell deine Frage

Willkommen bei der Mathelounge! Stell deine Frage einfach und kostenlos

x
Made by a lovely community